In a 45-45-90 right triangle, what is the ratio of the length of one leg to the length of the other leg? O A. sqrt2:1 O B. 1:1 Oc. C. 2:1 O D. 1:sqrt2​

Answers

Answer 1
B, they are the same length
Answer 2

Answer:

B. 1 : 1

Step-by-step explanation:

the triangle has the same size of legs (the same angles 45°)


Related Questions

What percent is represented by the shaded area?

Answers

Answer:91%

Step-by-step explanation:

PLSSS, NEED ANSWER. Find the midpoint of the line segment with end coordinates of (-2,-5 and 3,-2
). Give coordinates as decimals where apropriate​

Answers

Answer:

(0.5, -3.5)

Step-by-step explanation:

(x1 + x2/2,   y1 + y2/2)

(-2 + 3 / 2,   -5 - 2 / 2)

(1/2, -7/2)

On a coordinate plane, triangle A B C has points (negative 2, negative 1), (negative 2, negative 3), (negative 5, negative 1). Triangle ABC is translated 6 units to the right and 1 units up. What are the coordinates of C'? C'(4, –2) C'(4, –0) C'(4, 2) C'(1, 2)

Answers

Answer:

The answer is A

Step-by-step explanation:

C’(4, –2)

The coordinates of C' are (1, 0)

What is translation?

A translation moves a shape up, down or from side to side, but it does not change its appearance in any other way.

Given that, On a coordinate plane, triangle A B C has points (-2, -1), (-2, -3), (-5, -1).

When Triangle ABC is translated 6 units to the right and 1 units up.

We know that rule for translating b unit up and a unit right is

(x, y) → (x+a, y+b)

Hence, the coordinates for C' =

(-5, -1) = (-5+6, -1+1) = (1, 0)

Hence, the coordinates of C' are (1, 0)

For more references on translation, click;

https://brainly.com/question/12463306

#SPJ6

Which fraction converts to a repeating decimal number?
CA.
1
12
B.
718
C.
127
27
D.
E.
6
10
Reset

Answers

Answer: A.

Step-by-step explanation:

Data: Fraction that turning into a repeating decimal number=x

Only step: Divide all the fractions, 1/12, 7/8, 14/25, 17/20, 6/10

Explanation: The only way to find which fraction turns into a repeating decimal is by dividing all the fractions, this can be done in any order but for this problem, lets start with 1/12 which, when divided, turns into 0.083... which is a repeating decimal

With that being said, the answer would be A.(1/12)

I hope this helps(Mark brainliest if you'd like to)

Surface area of a cube please HELP HELP HELP

Answers

Answer:

The surface area of the cube is 294 m^2

Step-by-step explanation:

The formula for the surface area of a cube is:

(a^2) 6

Explanation behind formula:

a represents the side length. a^2 is the area of one face of the cube. Multiply that by 6 because there are 6 faces on a cube.

(Surface area- the area of each face of a figure)

Use formula with the given:

(7^2) 6 = 294

Surface area is measured in square meters.

(meters in this situation)

Therefore, the surface area of the cube is

294 m^2

Hope this helps!

Answer:

294

Step-by-step explanation:

formula is = 6(side length)^2

6x(7)^2

=294

Students are asked to estimate the number of gumballs in a jar. Sam says there are 228 gumballs. In actuality, there are 240 gumballs. What is the percent error

Answers

Answer:

5%

Step-by-step explanation:

Percent error = (actual - estimated) / actual x 100

(240 - 228) / 240 x 100 = 5%

Given the equation −4square root of x minus 3 = -12, solve for x and identify if it is an extraneous solution.
A. x = 0, solution is extraneous
B. x = 0, solution is not extraneous
C. x = 12, solution is extraneous
D. x = 12, solution is not extraneous

Answers

Answer:

D.    x = 12, solution is not extraneous

Step-by-step explanation:

-4√(x - 3) = -12

Divide both sides by -4

√(x - 3) = 3

Square both sides

x - 3 = 9

add 3 to both sides

x = 12

To see if it's extraneous plug into original problem

-4√(12 - 3) = -12

-4√(9) = -12

-4 * 3 = -12

-12 = -12

not extraneous

Will mark brainliest

Answers

Answer:

the answer would be 270

Step-by-step explanation:

i can explain later:

since x = 5, you plug it in f = 4x +7, and you get f = 27 and since 1/2x is in the denominator, it will flip over and become 2x. 2x is 10, so you multiply f by g and get 270 which is if f/g(5) is the question.

In the diagram below of triangle PQS, S is the midpoint of PR and T is the midpoint of QR. If ST = 5x - 22, and PQ = 3x + 19, what is the measurement of PQ?

Answers

Answer:

PQ = 46

Step-by-step explanation:

The midsegment ST is half the length of the third side PQ , that is

ST = [tex]\frac{1}{2}[/tex] PQ , so

5x - 22 = [tex]\frac{1}{2}[/tex] (3x + 19) ← multiply both sides by 2 to clear the fraction

10x - 44 = 3x + 19 ( subtract 3x from both sides )

7x - 44 = 19 ( add 44 to both sides )

7x = 63 ( divide both sides by 7 )

x = 9

Then

PQ = 3x + 19 = 3(9) + 19 = 27 + 19 = 46

Which of the following is the proper name for the figure below?
T
M
A
y
X
Ο Α. ΔΑΤΧ
Ο Β. ΔΑΤΜ
Ο C. ΔΑΥΜ
Ο D. ΔΑΥΧ

Answers

Answer:

A

Step-by-step explanation:

i don't know the real answer but i have learned it before so it's an educated guess

give your answer in detail plzz ​

Answers

Answer:

60% did not germinate.

Step-by-step explanation:

100% = 5

how many percent are 2 of of these 5 ?

2 = 2/5 × 5 = 2/5 × 100% = 40%

all the other seeds are then the rest of 100%.

100% - 40% = 60%

=>

60% of the seeds did not germinate.

but 40% did.

4.2 To buy a new car is not a very good investment. In the first year the car will
decrease in value by 25%.
4.2.1
If the car costs R200 000, what will it be worth after 1 year?

Answers

Given:

In the first year, the car will decrease in value by 25%.

The cost of a car is Rs. 200,000.

To find:

The worth of the car after 1 year.

Solution:

The cost of a car is Rs. 200,000 and in the first year the car will decrease in value by 25%. So, the worth of the car after 1 year is:

[tex]Worth=200000-\dfrac{25}{100}\times 200000[/tex]

[tex]Worth=200000-\dfrac{1}{4}\times 200000[/tex]

[tex]Worth=200000-50000[/tex]

[tex]Worth=150000[/tex]

Therefore, the worth of the car after 1 year is Rs. 150,000.

help 0-0-0-0-0-0-0-0-0-0-0-0-0-0

Answers

Answer:

(12,0)

Step-by-step explanation:

So whenever you are asked for an "intercept" it is the point where a line crosses the axis. The x intercept is where the line crosses at the x axis (horizontal line on the graph) and the y intercept is where the y axis crosses (vertical axis).

So this question is really asking where is the point when y = 0

The x values in the chart are 5 apart-- 22 +5 = 27, 27+ 5 = 32

The y values are 18 apart --> 36 + 18 = 54, 54+ 18 = 72

Make a little chart where y = 0 (this is where the x crosses) and you can see

there is a point at (17, 18) and then (12, 0) and that point is the x intercept.

Answer:

(12, 0)

Step-by-step explanation:

the x-intercept is the point, where y=0.

so, the second coordinate is therefore 0, no matter what.

now, at what x is that ?

let's find the line function.

a line is

y = a×x + b

let's use the first point (22, 36)

36 = a×22 + b

a = (36 - b)/22

let's use the second point (27, 54)

54 = a×27 + b

now we use the first expression describing a in terms of b and insert that into the second expression.

54 = 27×(36 - b)/22 + b

54×22 = 27×(36 - b) + 22b = 972 - 27b + 22b

1188 = 972 - 5b

216 = -5b

b = -43.2

=>

a = (36 - -43.2)/22 = (36 + 43.2)/22 =79.2/22 = 3.6

our line is

y = 3.6x - 43.2

so, now back to the question, at what x is y=0 ?

0 = 3.6x - 43.2

3.6x = 43.2

x = 12

Can someone helpppppppp

Answers

Answer:

f(-2) = -3

Step-by-step explanation:

When x = -2 we want the value of y

We use the closed circle value ( open circle does not have a value)

f(-2) = -3

A circle has a radius of 6cm. What is the area of a sector subtended by an angle of 30 degrees

Answers

Answer:

1/12 of the area of the circle

Step-by-step explanation:

Since the total angle measure of a circle is 360 degrees, a 30 degree sector is 30/360=1/12

i have no idea what to do can someone pls help me quick

Answers

The correct answer is D. I found this by plugging in zero for X into each equation until I got 1/4 for a Y
The correct answer is d

if 2x + 3y = 12 and xy = 6, find the value of 8x^3 + 27y^3​

Answers

Answer:

The value of [tex]8\cdot x^{3} + 27\cdot y^{3}[/tex] is 432.

Step-by-step explanation:

Let be the following system of equations:

[tex]2\cdot x + 3\cdot y = 12[/tex] (1)

[tex]x\cdot y = 6[/tex] (2)

Then, we solve both for [tex]x[/tex] and [tex]y[/tex]:

From (1):

[tex]2\cdot x + 3\cdot y = 12[/tex]

[tex]2\cdot x = 12- 3\cdot y[/tex]

[tex]x = 6 - \frac{3}{2}\cdot y[/tex]

(1) in (2):

[tex]\left(6-\frac{3}{2}\cdot y \right)\cdot y = 6[/tex]

[tex]6\cdot y-\frac{3}{2}\cdot y^{2} = 6[/tex]

[tex]\frac{3}{2}\cdot y^{2}-6\cdot y + 6 = 0[/tex]

The roots of the polynomial are determined by the Quadratic Formula:

[tex]y_{1} = y_{2} = 2[/tex]

By (1):

[tex]x = 6 - \frac{3}{2}\cdot (2)[/tex]

[tex]x = 3[/tex]

If we know that [tex]x = 3[/tex] and [tex]y = 2[/tex], then the final value is:

[tex]z = 8\cdot x^{3}+27\cdot y^{3}[/tex]

[tex]z = 8\cdot 3^{3}+27\cdot 2^{3}[/tex]

[tex]z = 432[/tex]

The value of [tex]8\cdot x^{3} + 27\cdot y^{3}[/tex] is 432.

I don't understand help plz​

Answers

It’s a and b are the correct answer
A and b are the correct answers

Convert the following decimal to fraction 3.04

Answers

Answer:

3 1/25

Step-by-step explanation:

3.04

Do the decimal part first

.04 = 4/100

Reduce - cancel out 4

1/25

Put back the whole number

3 1/25

Answer:

3 [tex]\frac{1}{25}[/tex]

Step-by-step explanation:

I agree with the previous answer :

You need to do it part by part.

0.04 is equal to [tex]\frac{4}{100}[/tex]

[tex]\frac{4}{100}[/tex] simplified (we divide by 4 with both the denomantor and the numerator.) =

[tex]\frac{1}{25}[/tex]

And we still have those 3 wholes that we had in the beginning.

Now we add.

3 + [tex]\frac{1}{25}[/tex] = 3 [tex]\frac{1}{25}[/tex]

1011+1111=in binary answer, anyone that answer me first I will follow u​

Answers

Answer:

Step-by-step explanation:

answer is   [tex](11010)_{2}[/tex]

Pls help will mark brainliest and 30 points

Answers

1. Answer is option 3

2. Answer is option 3

Answer:

for #1 it's the 3rd one and for #2 it's the 4th one (2.8 meters)

pls help me..... the question is in the attachment.... thank you


Answers

Answer:

I added some attachments as well, I hoped I helped.

Evaluate e^4 to one decimal place

Answers

Answer:

The answer is going to be the general form for e^x = 1 + x/(1!) + + x^2/(2!) + x^3/(3!) +.....,. till infinity so : e^4 = 1 + 4/1 + 16/ 2 + 64/6 + 256/24 + 1024/ 120 + 4096/720 + 16384/5040 +...till the fraction is<0.1 = 54.6

Dylan is saving money to buy a new bike that costs $195. He has $25 already saved and plans to save $40 each week. Which equation can be used to find the number of weeks he will need to save money to buy the bike?

Answers

Answer:

25+40x=195

Step-by-step explanation:

If he already has $25, you need to add that to 40x (40 times the amount of weeks it takes to get to 195, added with 25).

Simplify the expression....​

Answers

Answer:

−3x^2+2x /x−2

Step-by-step explanation:

4x−9x^3/ 3x^2−4x−4

=  −9x^3+4x /3x^2−4x−4

=  x(−3x+2)(3x+2) /(3x+2)(x−2)

=  −3x^2+2x /x−2

Which expression is equivalent to 10k + 17 - 7j - 18 - 11k?
-8jk - 1
-7j - k - 1
-7j + k + 1
-8j - k



PLEASE HURRY !!!!
20 POINTS!!!

Answers

Answer:

-7j-k-1

Step-by-step explanation:

Answer:

C

Step-by-step explanation:

An equivalent expression is an expression which is equal to 10k + 17 - 7j - 18 - 11k. You can expand or simplify an expression to make an equivalent expression. Combine like terms to form a new equivalent expression. Like terms are terms which have the same base or variable.

10k + 17 - 7j - 18 - 11k

10k -11k + 17 - 18 - 7j                Move to group like terms together.

-k - 1 - 7j

The Jenkins family's monthly budget is
shown in the circle graph. The family has a
monthly income of $4,800. How much
money do they spend on transportation
each month?
Emergency fund
5%
Transportation
5%
Housing
30%
Medical
22%
Food
15%
Clothing
6%
Entertainment
7%
Savings
10%​

Answers

Answer:

$240

Step-by-step explanation:

They spend 5% of their monthly income on transportation, so you want to work out 5% of 4800.

An easy trick with 5% is to divide by 10 (drop the ending 0) and then divide by 2. This would be 4800/10/2, or 480/2, which gives 240.

**This content involves using percentages which you may wish to revise.

If the first three Fibonacci numbers are given as x, = 1, X2 = 1 and x3 = 2,
what is the value of n for which X, + Xp = 55?

Answers

Can you please help me with this question

Answer:

n=8

Step-by-step explanation:

Using the following image, complete the statement below. I got all the answers I need the ones that is blank

Answers

Answer:

DE is congruent to BE , ED is congruent is EB and the last one is midpoint.

Step-by-step explanation:

joey is going shopping for a new pair of sneakers. He finds a pair that have an original price of $155. They are on sale today for 30% off. How much does Joey pay for the sneakers including 8% sales tax?

Answers

Answer:, Joey will pay $117.18 for sneakers.

Step-by-step explanation:

Given: original price = $155

Discount rate = 30%

Tax rate = 8%

Price after discount = Original price - (Discount) x (original price)

[tex]= 155-0.30\times 155\\\\=155-46.5\\\\=\$\ 108.5[/tex]

Tax = Tax rate x (Price after discount)

[tex]= 0.08 \times 108.5[/tex]

= $ 8.68

Final price for sneakers = Price after discount + Tax

= $ (108.5+8.68)

= $ 117.18

Hence

Other Questions
in a sale, normal prices are reduced by 10%nathalie bought a pair of shoes in the sale for 54what was the original price of the shoes PRIMARY SOURCEA You went down one step even from the foul area into the cellar in which a family of human beings lived. It was very dark inside. The window-panes many of them were broken and stuffed with rags....the smell was so fetid [foul] as almost to knock the two men down.... and to see three or four little children rolling on the damp, nay wet brick floor, through which the stagnant, filthy moisture of the street oozed up. .they began to penetrate the thick darkness of the place, ELIZABETH GASKELL, Mary Barton 1. What is the value of the digit 7 in the number 176 354? If a star is 5,699,999,999,999,999 meters from earth, how long does it take light to travel from earth to the star? express as a single faction [tex] \frac{x} {4} + \frac{x}{4} [/tex] Consider the following events that take place when rip currents occur.A. Waves travel to the beach.B. Waves are trapped by the sandbars.C. Waves reach the shore and go back to the ocean.D. Waves speed up and flow between the sandbars.E. Waves are broken by the sandbars.Which list shows the order of events in the production of rip currents? i need help w this pls PLEASE HELP IMAGE IS BELOW!! In a Nitrogen metabolism study, the following data were obtained over the test period:Nitrogen consumed 175 gramsTotal Fecal N 35 gramsMetabolic Fecal N 10 gramsTotal Urinary N 80 gramsEndogenous Urinary N 10 gramsCalculate: a. Apparent Nitrogen digestibility (%)b. True Nitrogen digestibility (%)c. Nitrogen Balance (g)d. Apparent Nitrogen Retention (%) A soccer ball is kicked toward the goal. The height of the ball is modeled by the function h(t) = - 16t^2 + 48t where t equals the time in seconds and the h(t) represents the height of the ball at time t seconds. What is the axis of symmetry, and what does it represent? Help please!!! HELP ME PLEASE!!!Which 2 statements are true about this chemical reaction that forms acid rain? Lily's car has a fuel efficiency of 8 liters per 100 kilometers. What is the fuel efficiency of Lily's car in kilometers per liter? km L What kinds of activities would help to promote intrinsic motivation? How did William the Conqueror and King Henry II of England expand the monarchys power? Use the cause-effect graphic organizer to organize your ideas. On each line, describe an effect of the (sorry for incomplete question its what was written) Use algebra to solve 3x+4 = 1/xThe exact solutions are x= What did the wind ask the leaves to do Solve the system by substitution.y = -5xy= -7x 14 Draw a graph of the linear equations x 4y + 10 = 0 and x + 4y 10 = 0. Find the coordinates of the vertices of the triangle formed by these lines along with the X-axis Find the trigonometric ratio. Write your answer as a fraction in the simplest form Consider all four-digit numbers that can be made from the digits 0-8 (assume that numbers cannot start with 0). What is the probability of choosing a random number from this group that is less than or equal to 4000